Find sequence $(a_n)$ of positive reals such that $sum a_n$ diverges but $sum a_n/(ln(ln(n)))$ converges....












0












$begingroup$


I want to find an example of a sequence $a_n >0$ such that its series $sum a_n$ diverges, but when we consider $$sum_{i=2}^n frac{a_n}{ln(ln(n))}$$ then this series is in fact convergent.



Any idea what to start with, what motivation would you use to find such a sequence?










share|cite|improve this question











$endgroup$



closed as off-topic by amWhy, Nosrati, Saad, GNUSupporter 8964民主女神 地下教會, Crostul Dec 10 '18 at 14:17


This question appears to be off-topic. The users who voted to close gave this specific reason:


  • "This question is missing context or other details: Please provide additional context, which ideally explains why the question is relevant to you and our community. Some forms of context include: background and motivation, relevant definitions, source, possible strategies, your current progress, why the question is interesting or important, etc." – amWhy, Nosrati, Saad, GNUSupporter 8964民主女神 地下教會, Crostul

If this question can be reworded to fit the rules in the help center, please edit the question.
















  • $begingroup$
    Sorry. My bad. I've addes some essential info.
    $endgroup$
    – Filip Wichrowski
    Dec 10 '18 at 12:59










  • $begingroup$
    @FilipWichrowski: if we use the variable $n$, use $log$ instead of $ln$. of course, both are correct, but $log n$ looks fine than $ln n$
    $endgroup$
    – Chinnapparaj R
    Dec 10 '18 at 13:05










  • $begingroup$
    but $ln$ is the proper name for this $log$
    $endgroup$
    – Wesley Strik
    Dec 10 '18 at 13:32












  • $begingroup$
    what would we like to see added to make this question better?
    $endgroup$
    – Wesley Strik
    Dec 10 '18 at 16:39
















0












$begingroup$


I want to find an example of a sequence $a_n >0$ such that its series $sum a_n$ diverges, but when we consider $$sum_{i=2}^n frac{a_n}{ln(ln(n))}$$ then this series is in fact convergent.



Any idea what to start with, what motivation would you use to find such a sequence?










share|cite|improve this question











$endgroup$



closed as off-topic by amWhy, Nosrati, Saad, GNUSupporter 8964民主女神 地下教會, Crostul Dec 10 '18 at 14:17


This question appears to be off-topic. The users who voted to close gave this specific reason:


  • "This question is missing context or other details: Please provide additional context, which ideally explains why the question is relevant to you and our community. Some forms of context include: background and motivation, relevant definitions, source, possible strategies, your current progress, why the question is interesting or important, etc." – amWhy, Nosrati, Saad, GNUSupporter 8964民主女神 地下教會, Crostul

If this question can be reworded to fit the rules in the help center, please edit the question.
















  • $begingroup$
    Sorry. My bad. I've addes some essential info.
    $endgroup$
    – Filip Wichrowski
    Dec 10 '18 at 12:59










  • $begingroup$
    @FilipWichrowski: if we use the variable $n$, use $log$ instead of $ln$. of course, both are correct, but $log n$ looks fine than $ln n$
    $endgroup$
    – Chinnapparaj R
    Dec 10 '18 at 13:05










  • $begingroup$
    but $ln$ is the proper name for this $log$
    $endgroup$
    – Wesley Strik
    Dec 10 '18 at 13:32












  • $begingroup$
    what would we like to see added to make this question better?
    $endgroup$
    – Wesley Strik
    Dec 10 '18 at 16:39














0












0








0





$begingroup$


I want to find an example of a sequence $a_n >0$ such that its series $sum a_n$ diverges, but when we consider $$sum_{i=2}^n frac{a_n}{ln(ln(n))}$$ then this series is in fact convergent.



Any idea what to start with, what motivation would you use to find such a sequence?










share|cite|improve this question











$endgroup$




I want to find an example of a sequence $a_n >0$ such that its series $sum a_n$ diverges, but when we consider $$sum_{i=2}^n frac{a_n}{ln(ln(n))}$$ then this series is in fact convergent.



Any idea what to start with, what motivation would you use to find such a sequence?







sequences-and-series analysis convergence






share|cite|improve this question















share|cite|improve this question













share|cite|improve this question




share|cite|improve this question








edited Dec 16 '18 at 5:42









Brahadeesh

6,22742361




6,22742361










asked Dec 10 '18 at 12:45









Filip WichrowskiFilip Wichrowski

206




206




closed as off-topic by amWhy, Nosrati, Saad, GNUSupporter 8964民主女神 地下教會, Crostul Dec 10 '18 at 14:17


This question appears to be off-topic. The users who voted to close gave this specific reason:


  • "This question is missing context or other details: Please provide additional context, which ideally explains why the question is relevant to you and our community. Some forms of context include: background and motivation, relevant definitions, source, possible strategies, your current progress, why the question is interesting or important, etc." – amWhy, Nosrati, Saad, GNUSupporter 8964民主女神 地下教會, Crostul

If this question can be reworded to fit the rules in the help center, please edit the question.







closed as off-topic by amWhy, Nosrati, Saad, GNUSupporter 8964民主女神 地下教會, Crostul Dec 10 '18 at 14:17


This question appears to be off-topic. The users who voted to close gave this specific reason:


  • "This question is missing context or other details: Please provide additional context, which ideally explains why the question is relevant to you and our community. Some forms of context include: background and motivation, relevant definitions, source, possible strategies, your current progress, why the question is interesting or important, etc." – amWhy, Nosrati, Saad, GNUSupporter 8964民主女神 地下教會, Crostul

If this question can be reworded to fit the rules in the help center, please edit the question.












  • $begingroup$
    Sorry. My bad. I've addes some essential info.
    $endgroup$
    – Filip Wichrowski
    Dec 10 '18 at 12:59










  • $begingroup$
    @FilipWichrowski: if we use the variable $n$, use $log$ instead of $ln$. of course, both are correct, but $log n$ looks fine than $ln n$
    $endgroup$
    – Chinnapparaj R
    Dec 10 '18 at 13:05










  • $begingroup$
    but $ln$ is the proper name for this $log$
    $endgroup$
    – Wesley Strik
    Dec 10 '18 at 13:32












  • $begingroup$
    what would we like to see added to make this question better?
    $endgroup$
    – Wesley Strik
    Dec 10 '18 at 16:39


















  • $begingroup$
    Sorry. My bad. I've addes some essential info.
    $endgroup$
    – Filip Wichrowski
    Dec 10 '18 at 12:59










  • $begingroup$
    @FilipWichrowski: if we use the variable $n$, use $log$ instead of $ln$. of course, both are correct, but $log n$ looks fine than $ln n$
    $endgroup$
    – Chinnapparaj R
    Dec 10 '18 at 13:05










  • $begingroup$
    but $ln$ is the proper name for this $log$
    $endgroup$
    – Wesley Strik
    Dec 10 '18 at 13:32












  • $begingroup$
    what would we like to see added to make this question better?
    $endgroup$
    – Wesley Strik
    Dec 10 '18 at 16:39
















$begingroup$
Sorry. My bad. I've addes some essential info.
$endgroup$
– Filip Wichrowski
Dec 10 '18 at 12:59




$begingroup$
Sorry. My bad. I've addes some essential info.
$endgroup$
– Filip Wichrowski
Dec 10 '18 at 12:59












$begingroup$
@FilipWichrowski: if we use the variable $n$, use $log$ instead of $ln$. of course, both are correct, but $log n$ looks fine than $ln n$
$endgroup$
– Chinnapparaj R
Dec 10 '18 at 13:05




$begingroup$
@FilipWichrowski: if we use the variable $n$, use $log$ instead of $ln$. of course, both are correct, but $log n$ looks fine than $ln n$
$endgroup$
– Chinnapparaj R
Dec 10 '18 at 13:05












$begingroup$
but $ln$ is the proper name for this $log$
$endgroup$
– Wesley Strik
Dec 10 '18 at 13:32






$begingroup$
but $ln$ is the proper name for this $log$
$endgroup$
– Wesley Strik
Dec 10 '18 at 13:32














$begingroup$
what would we like to see added to make this question better?
$endgroup$
– Wesley Strik
Dec 10 '18 at 16:39




$begingroup$
what would we like to see added to make this question better?
$endgroup$
– Wesley Strik
Dec 10 '18 at 16:39










1 Answer
1






active

oldest

votes


















5












$begingroup$

Take $$a_n=frac{1}{(n, ln n) ln( ln n)}$$ Then $$sum_{n=3}^infty a_n= sum_{n=3}^infty frac{1}{(n ,ln n) ln( ln n)} $$ diverges , but $$sum_{n=3}^infty frac{a_n}{ln(ln n)}= sum_{n=3}^inftyfrac{1}{(n,ln n) ln( ln n)}. frac{1}{ln(ln n)}=sum_{n=3}^infty frac{1}{n ,ln n [ln (ln n)]^2}< infty$$





The convergence and divergence are follows from:



Theorem 1: Suppose $a_1 geq a_2 geq cdots geq 0. $ Then $sum a_n < infty iff sum 2^k a_{2^k} < infty$



Theorem 2: If $p>1$, then $$sum frac{1}{n, (ln n)^p} $$ converges, and diverges otherwise






share|cite|improve this answer











$endgroup$









  • 1




    $begingroup$
    What led you to this form of $a_n$? And how to prove that $sum a_n = +inf$?
    $endgroup$
    – Filip Wichrowski
    Dec 10 '18 at 13:05








  • 1




    $begingroup$
    I suggest to use Cauchy condensation test
    $endgroup$
    – gimusi
    Dec 10 '18 at 13:12






  • 2




    $begingroup$
    @gimusi: I already mention that in my answer.
    $endgroup$
    – Chinnapparaj R
    Dec 10 '18 at 13:15






  • 2




    $begingroup$
    @ChinnapparajR That's fine sorry I didn't see that. Bye
    $endgroup$
    – gimusi
    Dec 10 '18 at 13:21






  • 1




    $begingroup$
    If this answers the question, I would say it's nice to accept it as an answer.
    $endgroup$
    – Wesley Strik
    Dec 10 '18 at 16:38


















1 Answer
1






active

oldest

votes








1 Answer
1






active

oldest

votes









active

oldest

votes






active

oldest

votes









5












$begingroup$

Take $$a_n=frac{1}{(n, ln n) ln( ln n)}$$ Then $$sum_{n=3}^infty a_n= sum_{n=3}^infty frac{1}{(n ,ln n) ln( ln n)} $$ diverges , but $$sum_{n=3}^infty frac{a_n}{ln(ln n)}= sum_{n=3}^inftyfrac{1}{(n,ln n) ln( ln n)}. frac{1}{ln(ln n)}=sum_{n=3}^infty frac{1}{n ,ln n [ln (ln n)]^2}< infty$$





The convergence and divergence are follows from:



Theorem 1: Suppose $a_1 geq a_2 geq cdots geq 0. $ Then $sum a_n < infty iff sum 2^k a_{2^k} < infty$



Theorem 2: If $p>1$, then $$sum frac{1}{n, (ln n)^p} $$ converges, and diverges otherwise






share|cite|improve this answer











$endgroup$









  • 1




    $begingroup$
    What led you to this form of $a_n$? And how to prove that $sum a_n = +inf$?
    $endgroup$
    – Filip Wichrowski
    Dec 10 '18 at 13:05








  • 1




    $begingroup$
    I suggest to use Cauchy condensation test
    $endgroup$
    – gimusi
    Dec 10 '18 at 13:12






  • 2




    $begingroup$
    @gimusi: I already mention that in my answer.
    $endgroup$
    – Chinnapparaj R
    Dec 10 '18 at 13:15






  • 2




    $begingroup$
    @ChinnapparajR That's fine sorry I didn't see that. Bye
    $endgroup$
    – gimusi
    Dec 10 '18 at 13:21






  • 1




    $begingroup$
    If this answers the question, I would say it's nice to accept it as an answer.
    $endgroup$
    – Wesley Strik
    Dec 10 '18 at 16:38
















5












$begingroup$

Take $$a_n=frac{1}{(n, ln n) ln( ln n)}$$ Then $$sum_{n=3}^infty a_n= sum_{n=3}^infty frac{1}{(n ,ln n) ln( ln n)} $$ diverges , but $$sum_{n=3}^infty frac{a_n}{ln(ln n)}= sum_{n=3}^inftyfrac{1}{(n,ln n) ln( ln n)}. frac{1}{ln(ln n)}=sum_{n=3}^infty frac{1}{n ,ln n [ln (ln n)]^2}< infty$$





The convergence and divergence are follows from:



Theorem 1: Suppose $a_1 geq a_2 geq cdots geq 0. $ Then $sum a_n < infty iff sum 2^k a_{2^k} < infty$



Theorem 2: If $p>1$, then $$sum frac{1}{n, (ln n)^p} $$ converges, and diverges otherwise






share|cite|improve this answer











$endgroup$









  • 1




    $begingroup$
    What led you to this form of $a_n$? And how to prove that $sum a_n = +inf$?
    $endgroup$
    – Filip Wichrowski
    Dec 10 '18 at 13:05








  • 1




    $begingroup$
    I suggest to use Cauchy condensation test
    $endgroup$
    – gimusi
    Dec 10 '18 at 13:12






  • 2




    $begingroup$
    @gimusi: I already mention that in my answer.
    $endgroup$
    – Chinnapparaj R
    Dec 10 '18 at 13:15






  • 2




    $begingroup$
    @ChinnapparajR That's fine sorry I didn't see that. Bye
    $endgroup$
    – gimusi
    Dec 10 '18 at 13:21






  • 1




    $begingroup$
    If this answers the question, I would say it's nice to accept it as an answer.
    $endgroup$
    – Wesley Strik
    Dec 10 '18 at 16:38














5












5








5





$begingroup$

Take $$a_n=frac{1}{(n, ln n) ln( ln n)}$$ Then $$sum_{n=3}^infty a_n= sum_{n=3}^infty frac{1}{(n ,ln n) ln( ln n)} $$ diverges , but $$sum_{n=3}^infty frac{a_n}{ln(ln n)}= sum_{n=3}^inftyfrac{1}{(n,ln n) ln( ln n)}. frac{1}{ln(ln n)}=sum_{n=3}^infty frac{1}{n ,ln n [ln (ln n)]^2}< infty$$





The convergence and divergence are follows from:



Theorem 1: Suppose $a_1 geq a_2 geq cdots geq 0. $ Then $sum a_n < infty iff sum 2^k a_{2^k} < infty$



Theorem 2: If $p>1$, then $$sum frac{1}{n, (ln n)^p} $$ converges, and diverges otherwise






share|cite|improve this answer











$endgroup$



Take $$a_n=frac{1}{(n, ln n) ln( ln n)}$$ Then $$sum_{n=3}^infty a_n= sum_{n=3}^infty frac{1}{(n ,ln n) ln( ln n)} $$ diverges , but $$sum_{n=3}^infty frac{a_n}{ln(ln n)}= sum_{n=3}^inftyfrac{1}{(n,ln n) ln( ln n)}. frac{1}{ln(ln n)}=sum_{n=3}^infty frac{1}{n ,ln n [ln (ln n)]^2}< infty$$





The convergence and divergence are follows from:



Theorem 1: Suppose $a_1 geq a_2 geq cdots geq 0. $ Then $sum a_n < infty iff sum 2^k a_{2^k} < infty$



Theorem 2: If $p>1$, then $$sum frac{1}{n, (ln n)^p} $$ converges, and diverges otherwise







share|cite|improve this answer














share|cite|improve this answer



share|cite|improve this answer








edited Dec 11 '18 at 15:02









amWhy

1




1










answered Dec 10 '18 at 13:00









Chinnapparaj RChinnapparaj R

5,4161828




5,4161828








  • 1




    $begingroup$
    What led you to this form of $a_n$? And how to prove that $sum a_n = +inf$?
    $endgroup$
    – Filip Wichrowski
    Dec 10 '18 at 13:05








  • 1




    $begingroup$
    I suggest to use Cauchy condensation test
    $endgroup$
    – gimusi
    Dec 10 '18 at 13:12






  • 2




    $begingroup$
    @gimusi: I already mention that in my answer.
    $endgroup$
    – Chinnapparaj R
    Dec 10 '18 at 13:15






  • 2




    $begingroup$
    @ChinnapparajR That's fine sorry I didn't see that. Bye
    $endgroup$
    – gimusi
    Dec 10 '18 at 13:21






  • 1




    $begingroup$
    If this answers the question, I would say it's nice to accept it as an answer.
    $endgroup$
    – Wesley Strik
    Dec 10 '18 at 16:38














  • 1




    $begingroup$
    What led you to this form of $a_n$? And how to prove that $sum a_n = +inf$?
    $endgroup$
    – Filip Wichrowski
    Dec 10 '18 at 13:05








  • 1




    $begingroup$
    I suggest to use Cauchy condensation test
    $endgroup$
    – gimusi
    Dec 10 '18 at 13:12






  • 2




    $begingroup$
    @gimusi: I already mention that in my answer.
    $endgroup$
    – Chinnapparaj R
    Dec 10 '18 at 13:15






  • 2




    $begingroup$
    @ChinnapparajR That's fine sorry I didn't see that. Bye
    $endgroup$
    – gimusi
    Dec 10 '18 at 13:21






  • 1




    $begingroup$
    If this answers the question, I would say it's nice to accept it as an answer.
    $endgroup$
    – Wesley Strik
    Dec 10 '18 at 16:38








1




1




$begingroup$
What led you to this form of $a_n$? And how to prove that $sum a_n = +inf$?
$endgroup$
– Filip Wichrowski
Dec 10 '18 at 13:05






$begingroup$
What led you to this form of $a_n$? And how to prove that $sum a_n = +inf$?
$endgroup$
– Filip Wichrowski
Dec 10 '18 at 13:05






1




1




$begingroup$
I suggest to use Cauchy condensation test
$endgroup$
– gimusi
Dec 10 '18 at 13:12




$begingroup$
I suggest to use Cauchy condensation test
$endgroup$
– gimusi
Dec 10 '18 at 13:12




2




2




$begingroup$
@gimusi: I already mention that in my answer.
$endgroup$
– Chinnapparaj R
Dec 10 '18 at 13:15




$begingroup$
@gimusi: I already mention that in my answer.
$endgroup$
– Chinnapparaj R
Dec 10 '18 at 13:15




2




2




$begingroup$
@ChinnapparajR That's fine sorry I didn't see that. Bye
$endgroup$
– gimusi
Dec 10 '18 at 13:21




$begingroup$
@ChinnapparajR That's fine sorry I didn't see that. Bye
$endgroup$
– gimusi
Dec 10 '18 at 13:21




1




1




$begingroup$
If this answers the question, I would say it's nice to accept it as an answer.
$endgroup$
– Wesley Strik
Dec 10 '18 at 16:38




$begingroup$
If this answers the question, I would say it's nice to accept it as an answer.
$endgroup$
– Wesley Strik
Dec 10 '18 at 16:38



Popular posts from this blog

To store a contact into the json file from server.js file using a class in NodeJS

Redirect URL with Chrome Remote Debugging Android Devices

Dieringhausen